Ise synthetic division to find the result when x^(4)+8x^(3)+16x^(2)-x-18 is ivided by x+3. If there is a remainder, express the result in the form (x)+(r(x))/(b(x))

Answers

Answer 1

The result of dividing x⁴ + 8x³ + 16x² - x - 18 by x + 3 is (x³ + 5x² + x - 4).

To perform synthetic division, we set up the problem as follows:

        -3 │ 1   8   16   -1   -18

           │

To start, we bring down the coefficient of the highest power term, which is 1:

        -3 │ 1   8   16   -1   -18

           │

           │  1

Next, we multiply -3 by the value we just brought down (1), and write the result below the next coefficient:

        -3 │ 1   8   16   -1   -18

           │     -3

           │  1

We then add the corresponding terms

        -3 │ 1   8   16   -1   -18

           │     -3

           │--------

           │  1   5

We repeat the process by multiplying -3 with the new value (5), and write the result below the next coefficient:

        -3 │ 1   8   16   -1   -18

           │     -3   -15

           │--------

           │  1   5    1

We continue with the process:

        -3 │ 1   8   16   -1   -18

           │     -3   -15    -3

           │-----------------

           │  1   5    1    -4

The resulting expression after performing synthetic division is 1x³ + 5x² + x - 4. There is no remainder in this case.

Therefore, the result of dividing x⁴ + 8x³ + 16x² - x - 18 by x + 3 is (x³ + 5x² + x - 4).

To know more about dividing click here :

https://brainly.com/question/11850611

#SPJ4


Related Questions

The five number summary of a data set was found to be: \[ 46,54,60,65,70 \] What is the interquartile range?

Answers

The interquartile range for the given data set is 17.5.

Given, The five number summary of a data set was found to be: \[ 46,54,60,65,70 \].

The interquartile range (IQR) can be calculated using the following formula:

IQR = Q3 - Q1,

where Q3 represents the third quartile, and Q1 represents the first quartile.

To find the interquartile range (IQR), let us first find the first quartile and the third quartile of the data set:

First Quartile (Q1):

Median of the lower half of the data set \[ 46, 54 \]

Median = (46 + 54) / 2 = 50

Third Quartile (Q3):

Median of the upper half of the data set \[ 65, 70 \]

Median = (65 + 70) / 2 = 67.5

Using the values obtained, we can now calculate the interquartile range (IQR) as follows:

IQR = Q3 - Q1

IQR = 67.5 - 50

IQR = 17.5

Therefore, the interquartile range for the given data set is 17.5.

Learn more about interquartile range: https://brainly.com/question/1210750

#SPJ11

Determine the coefficient of each term, 9x^(7)+x^(5)-3x^(3)+6 The coefficient of the term 9x^(7) is

Answers

The coefficient of the term 9x^7 is 9. In the given polynomial expression, the term 9x^7 represents the product of the coefficient (9) and the variable raised to the power of 7 (x^7).

In the polynomial expression 9x^7 + x^5 - 3x^3 + 6, each term consists of a coefficient and a variable raised to a certain power. The coefficient represents the numerical factor multiplied by the variable term. In the term 9x^7, the coefficient is 9. This means that the variable x is multiplied by 9 raised to the power of 7, resulting in 9x^7.

The coefficient of a term determines the scale or magnitude of that term within the polynomial expression. It indicates the amount by which the term contributes to the overall value of the expression. In this case, the coefficient of 9 in 9x^7 implies that the term 9x^7 has a greater impact on the polynomial's value compared to other terms, such as x^5, -3x^3, and 6.

To know more about coefficient refer here:

https://brainly.com/question/13431100

#SPJ11

For each of the following recurrences, sketch its recursion tree and guess a good asymptotic upper bound on its solution. Then use the substitution method to verify your answer.
a. T(n) = T(n/2) + n3
b. T(n) = 4T(n/3) + n
c. T(n) = 4T(n/2) + n
d. T(n) = 3T (n -1) + 1

Answers

The asymptotic upper bounds for the given recurrence relations are: (a) O(n^3 * log(n)), (b) O(n^log_3(4)), (c) O(n^2 * log(n)), and (d) O(n). The substitution method can be used to verify these bounds.

(a) For the recurrence relation T(n) = T(n/2) + n^3, the recursion tree will have log(n) levels with n^3 work done at each level. Therefore, the total work done can be approximated as O(n^3 * log(n)). This can be verified using the substitution method.

(b) In the recurrence relation T(n) = 4T(n/3) + n, the recursion tree will have log_3(n) levels with n work done at each level. Therefore, the total work done can be approximated as O(n^log_3(4)) using the Master Theorem. This can also be verified using the substitution method.

(c) The recurrence relation T(n) = 4T(n/2) + n will have a recursion tree with log_2(n) levels and n work done at each level. Hence, the total work done can be approximated as O(n^2 * log(n)) using the Master Theorem. This can be verified using the substitution method.

(d) The recurrence relation T(n) = 3T(n-1) + 1 will result in a recursion tree with n levels and constant work done at each level. Therefore, the total work done can be approximated as O(n). This can be verified using the substitution method.

Visit here to learn more about substitution method:

brainly.com/question/22340165

#SPJ11

Suppose that we have a bulbs box containing 60 bulbs, of which 13 are defective. 2 bulbs are slected at random, with replacement from the box (Round your answer to three decimals) A) Find the probability that both bulbs are defective. B) Find the probability that atleast one of them is defective.

Answers

a) The probability that both bulbs are defective is approximately 0.047.

b) The probability that at least one of the bulbs is defective is approximately 0.386. These probabilities were calculated using the binomial distribution with n = 2 and p = 13/60 for defective bulbs.

We can use the binomial distribution to solve this problem. Let X be the number of defective bulbs in a sample of size 2, with replacement. Then X follows a binomial distribution with n = 2 and p = 13/60 for defective bulbs.

a) The probability that both bulbs are defective is:

P(X = 2) = (2 choose 2) * (13/60)^2 * (47/60)^0

= 1 * (169/3600) * 1

= 169/3600

≈ 0.047

Therefore, the probability that both bulbs are defective is approximately 0.047.

b) The probability that at least one of the bulbs is defective is:

P(X ≥ 1) = 1 - P(X = 0)

= 1 - (2 choose 0) * (13/60)^0 * (47/60)^2

= 1 - 1 * 1 * (2209/3600)

= 1391/3600

≈ 0.386

Therefore, the probability that at least one of the bulbs is defective is approximately 0.386.

learn more about binomial distribution here

https://brainly.com/question/29137961

#SPJ11

A friend offers you a free ticket to a concert, which you decide to attend. The concert takes 4 hours and costs you $15 for transportation. If you had not attended the concert, you would have worked at your part-time job earning $15 per hour. What is the true cost of you attending the concert?

Answers

The true cost of you attending the concert is $60.

The correct answer for the given problem is as follows:

Opportunity cost is the true cost of you attending the concert.

The reason being, the person had to give up an alternative use of their time to attend the concert.

In the given situation, if the person had not attended the concert they would have worked at their part-time job earning $15 per hour.

Thus, the opportunity cost for attending the concert is equal to the amount of money you would have earned had you not gone to the concert.

So, the opportunity cost of attending the concert would be: $15/hour × 4 hours = $60

The true cost of you attending the concert is $60.

Know more about cost:

https://brainly.com/question/17120857

#SPJ11

Which of the following gives the equation of a circle of radius 22 and center at the point (-1,2)(-1,2)?

Answers

Step-by-step explanation:

Equation of a circle is

[tex](x - h) {}^{2} + (y - k) {}^{2} = {r}^{2} [/tex]

where (h,k) is the center

and the radius is r.

Here the center is (-1,2) and the radius is 22

[tex](x + 1) {}^{2} + (y - 2) {}^{2} = 484[/tex]

Suppose x is a normally distributed random variable with µ = 15 and σ = 2. Find each of the following probabilities.
a. P(x219) b. P(xs13) c. P(15.58 sxs 19.58) d. P(10.28 ≤x≤ 17.94)

Answers

A.  P(x > 19) is also approximately 0.0228.

B. P(x < 13) is also approximately 0.1587.

C. P(15.58 < x < 19.58) is also approximately 0.4893.

D. P(10.28 ≤ x ≤ 17.94) is also approximately 0.8226.

a. P(x>19):

We need to standardize the variable x using the z-score formula:

z = (x - µ) / σ

Substituting the values we get,

z = (19 - 15) / 2 = 2

Using a standard normal distribution table or calculator, we find that P(z > 2) is approximately 0.0228. Therefore, P(x > 19) is also approximately 0.0228.

b. P(x < 13):

Again, we use the z-score formula:

z = (x - µ) / σ

Substituting the values we get,

z = (13 - 15) / 2 = -1

Using a standard normal distribution table or calculator, we find that P(z < -1) is approximately 0.1587. Therefore, P(x < 13) is also approximately 0.1587.

c. P(15.58 < x < 19.58):

We need to standardize both values of x using the z-score formula:

z1 = (15.58 - 15) / 2 = 0.29

z2 = (19.58 - 15) / 2 = 2.29

Using a standard normal distribution table or calculator, we find that P(0 < z < 2.29) is approximately 0.9893 - 0.5 = 0.4893. Therefore, P(15.58 < x < 19.58) is also approximately 0.4893.

d. P(10.28 ≤ x ≤ 17.94):

We standardize both values of x using the z-score formula:

z1 = (10.28 - 15) / 2 = -2.36

z2 = (17.94 - 15) / 2 = 0.97

Using a standard normal distribution table or calculator, we find that P(-2.36 ≤ z ≤ 0.97) is approximately 0.8325 - 0.0099 = 0.8226. Therefore, P(10.28 ≤ x ≤ 17.94) is also approximately 0.8226.

Learn more about approximately from

https://brainly.com/question/27894163

#SPJ11

Sep 26,5:58:07PM Watch help video Find an expression which represents the difference when (5x+6y) is subtracted from (2x+7y) in simplest terms.

Answers

To find an expression that represents the difference when (5x + 6y) is subtracted from (2x + 7y), we need to subtract (5x + 6y) from (2x + 7y).

When we subtract (5x + 6y) from (2x + 7y), we get:(2x + 7y) - (5x + 6y) = 2x + 7y - 5x - 6yNow we can simplify the expression by combining like terms. The like terms are the x terms and the y terms, so we group them separately:2x - 5x + 7y - 6y = -3x + ySo the expression that represents the difference when (5x + 6y) is subtracted from (2x + 7y) in simplest terms is: -3x + y.Note: The expression -3x + y represents the difference of the terms 2x + 7y and 5x + 6y.

To know more about expression visit:

https://brainly.com/question/28170201

#SPJ11

6×7N −2×3N is divisible by 4 , for N≥1

Answers

To determine whether the expression 6×7N − 2×3N is divisible by 4 for N≥1, let's simplify the expression first:

6×7N − 2×3N = 42N - 6N = 36N.

Now we need to check whether 36N is divisible by 4 for N≥1.

We know that a number is divisible by 4 if its last two digits (in decimal representation) are divisible by 4.

In this case, we are dealing with a variable N, so we need to analyze the possibilities for the last two digits of N that would make 36N divisible by 4.

The last two digits of N can be 00, 01, 02, ..., 98, or 99. Let's consider each case:

1. N = 00: 36N = 36×00 = 0. Divisible by 4.

2. N = 01: 36N = 36×01 = 36. Not divisible by 4.

3. N = 02: 36N = 36×02 = 72. Not divisible by 4.

4. N = 03: 36N = 36×03 = 108. Divisible by 4.

5. N = 04: 36N = 36×04 = 144. Divisible by 4.

6. N = 05: 36N = 36×05 = 180. Divisible by 4.

7. N = 06: 36N = 36×06 = 216. Divisible by 4.

8. N = 07: 36N = 36×07 = 252. Divisible by 4.

9. N = 08: 36N = 36×08 = 288. Divisible by 4.

10. N = 09: 36N = 36×09 = 324. Divisible by 4.

From the analysis above, we can conclude that for N≥1, the expression 6×7N − 2×3N is divisible by 4.

Learn more about Decimal here:

https://brainly.com/question/30958821

#SPJ11

How can thee model be ued to determine 1. 42−0. 53? Enter your anwer in the boxe. You cannot ubtract 5 tenth from 4 tenth or 3 hundredth from 2 hundredth, o regroup one whole into 10 tenth and then regroup one tenth into 10 hundredth. There are now 0 whole, tenth, and hundredth. After removing 5 tenth and 3 hundredth, there are tenth and hundredth remaining. Therefore, the difference of 1. 42 and 0. 53 i

Answers

The difference between 1.42 and 0.53 is 0.37.

The model can be used to determine the difference between 1.42 and 0.53.

First, we start with 1 whole and 4 tenths (1.4) and represent it in the model. Next, we subtract 5 tenths (0.5) from 4 tenths (0.4). Since we cannot subtract directly, we need to regroup. We can regroup 1 whole into 10 tenths and then regroup 1 tenth into 10 hundredths. Now we have 10 tenths (1) and 40 hundredths (0.4).

Next, we subtract 3 hundredths (0.03) from 40 hundredths (0.4). This can be done directly since the place values match. Subtracting, we get 37 hundredths (0.37).

Therefore, the difference between 1.42 and 0.53 is 0.37.

To summarize, we regrouped to subtract 5 tenths from 4 tenths, and then subtracted 3 hundredths from 40 hundredths. The final answer is 0.37.

Learn more about subtraction using models :

https://brainly.com/question/32595757

#SPJ11

PV81-x²
where x represents the number of hundreds of canisters and p is the price, in dollars, of a single canister.
(a) If p = 7, find the corresponding value of x.
x=11

Answers

The corresponding value of x when p = 7 is x = 11.

Given the equation PV = 81 - x², where x represents the number of hundreds of canisters and p is the price of a single canister in dollars.

To find the corresponding value of x when p = 7, we substitute p = 7 into the equation:

7V = 81 - x²

Rearranging the equation:

x² = 81 - 7V

To find the corresponding value of x, we need to know the value of V. Without the specific value of V, we cannot determine the exact value of x.

However, if we are given additional information about V, we can substitute it into the equation and solve for x. In this case, if the value of V is such that 7V is equal to 81, then the equation becomes:

7V = 81 - x²

Since 7V is equal to 81, we have:

7(1) = 81 - x²

7 = 81 - x²

Rearranging the equation:

x² = 81 - 7

x² = 74

Taking the square root of both sides:

x = ±√74

Since x represents the number of hundreds of canisters, the value of x must be positive. Therefore, the corresponding value of x when p = 7 is x = √74, which is approximately equal to 8.60. However, it's important to note that without additional information about the value of V, we cannot determine the exact value of x.

Learn more about corresponding value here:

brainly.com/question/12682395

#SPJ11

Rachel gets a starting salavy of P^(6000) amonth, and an increase of perd annually. What will be her salary durieg the fifth year?

Answers

Rachel's salary during the fifth year will be P^(8316.15) per month. Rachel gets a starting salary of P^(6000) per month and an increase of p% annually.

We are required to calculate her salary during the fifth year. To calculate the salary during the fifth year, we need to find out the salary for each of the five years. The salary during the first year will be P^(6000), and the salary during the second year can be calculated as follows:

Salary after the first year = P^(6000) + P^(6000) × p/100

= P^(6000) × (1 + p/100)

Similarly, the salary during the third year will be: Salary after the second year = P^(6000) × (1 + p/100) + P^(6000) × (1 + p/100) × p/100

= P^(6000) × (1 + p/100)^2

Similarly, we can calculate the salaries for the fourth and fifth years as: Salary after the third year = P^(6000) × (1 + p/100)^3

Salary after the fourth year = P^(6000) × (1 + p/100)^4

Salary after the fifth year = P^(6000) × (1 + p/100)^5

Given that Rachel gets an increase of p% annually, we can use the compound interest formula to calculate the value of p as follows:

We know that P^(8316.15) = P^(6000) × (1 + p/100)^5

Taking the fifth root on both sides, we get:1 + p/100 = (P^(8316.15) / P^(6000))^(1/5)

Substituting the values, we get:1 + p/100 = (1.3817217)

The value of p can be calculated as follows: p/100 = 0.3817217p = 38.17217%

Thus, Rachel's salary during the fifth year will be P^(8316.15) per month, which is approximately P^(8316).

To know more about salary visit:

https://brainly.com/question/29105365

#SPJ11

How to plot the function 2x+1 and 3x ∧
2+2 for x=−10:1:10 on the same plot. x=−10:1:10;y1=2 ∗
x+1;y2=3 ∗
x. ∧
2+2;plot(x,y1,x,y2) x=−10:1:10;y1=2 ∗
x+1;y2=3 ∗
x,a ∧
2+2; plot( x,y1); hold on: plot( x,y2) x=−10:1:10;y1=2 ∗
x+1;y2=3 ∗
x. ∧
2+2;plot(x,y1); plot (x,y2) Both a and b What is the syntax for giving the tag to the x-axis of the plot xlabel('string') xlabel(string) titlex('string') labelx('string') What is the syntax for giving the heading to the plot title('string') titleplot(string) header('string') headerplot('string') For x=[ 1

2

3

] and y=[ 4

5

6], Divide the current figure in 2 rows and 3 columns and plot vector x versus vector y on the 2 row and 2 column position. Which of the below command will perform it. x=[123];y=[45 6]; subplot(2,3,1), plot(x,y) x=[123]:y=[45 6): subplot(2,3,4), plot (x,y) x=[123]:y=[456]; subplot(2,3,5), plot(x,y) x=[123];y=[456]; subplot(3,2,4), plot( (x,y) What is the syntax for giving the tag to the y-axis of the plot ylabel('string') ylabel(string) titley('string') labely('string')

Answers

To plot the function 2x+1 and 3x^2+2 for x = -10:1:10 on the same plot, we will use the following command:

x = -10:1:10;

y1 = 2*x + 1;

y2 = 3*x.^2 + 2;

plot(x, y1);

plot(x, y2)

This will plot both functions on the same graph.

To tag the x-axis of the plot, we can use the command `xlabel('string')`, and to tag the y-axis, we can use `ylabel('string')`.

Therefore, the syntax for giving the tag to the x-axis is `xlabel('string')`, and the syntax for giving the tag to the y-axis is `ylabel('string')`.

We can provide a heading to the plot using the command `title('string')`. Hence, the syntax for giving the heading to the plot is `title('string')`.

To plot vector x versus vector y in the 2nd row and 2nd column position, we use the command `subplot(2, 3, 4), plot(x, y)`. Therefore, the correct option is:

x = [123];

y = [456];

subplot(3, 2, 4);

plot(x, y).

To know more about command visit:

https://brainly.com/question/32329589

#SPJ11

If the area of a circle is 821 what is the radius

Answers

Answer: r≈16.17

Step-by-step explanation: r=A

π=821

π≈16.16578

Let X⊆R^d be a set of d+1 affinely independent points. Show that int(conv(X))=∅.
a) √(1/3)a³. √12a² : √2a b) √(27x³y^{5}) : √(1/3)xy
d) 3x.(√27x^{5} : √(1/3)x³)

Answers

We have proved that if X ⊆ R^d is a set of d+1 affinely independent points, then int(conv(X)) ≠ ∅.

Given that X ⊆ R^d is a set of d+1 affinely independent points, we need to prove that int(conv(X)) ≠ ∅.

Definition: A set of points in Euclidean space is said to be affinely independent if no point in the set can be represented as an affine combination of the remaining points in the set.

Solution:

In order to show that int(conv(X)) ≠ ∅, we need to prove that the interior of the convex hull of the given set X is not an empty set. That is, there must exist a point that is interior to the convex hull of X.

Let X = {x_1, x_2, ..., x_{d+1}} be the set of d+1 affinely independent points in R^d. The convex hull of X is defined as the set of all convex combinations of the points in X. Hence, the convex hull of X is given by:

conv(X) = {t_1 x_1 + t_2 x_2 + ... + t_{d+1} x_{d+1} | t_1, t_2, ..., t_{d+1} ≥ 0 and t_1 + t_2 + ... + t_{d+1} = 1}

Now, let us consider the vector v = (1, 1, ..., 1) ∈ R^{d+1}. Note that the sum of the components of v is (d+1), which is equal to the number of points in X. Hence, we can write v as a convex combination of the points in X as follows:

v = (d+1)/∑i=1^{d+1} t_i (x_i)

where t_i = 1/(d+1) for all i ∈ {1, 2, ..., d+1}.

Note that t_i > 0 for all i and t_1 + t_2 + ... + t_{d+1} = 1, which satisfies the definition of a convex combination. Also, we have ∑i=1^{d+1} t_i = 1, which implies that v is in the convex hull of X. Hence, v ∈ conv(X).

Now, let us show that v is an interior point of conv(X). For this, we need to find an ε > 0 such that the ε-ball around v is completely contained in conv(X). Let ε = 1/(d+1). Then, for any point u in the ε-ball around v, we have:

|t_i - 1/(d+1)| ≤ ε for all i ∈ {1, 2, ..., d+1}

Hence, we have t_i ≥ ε > 0 for all i ∈ {1, 2, ..., d+1}. Also, we have:

∑i=1^{d+1} t_i = 1 + (d+1)(-1/(d+1)) = 0

which implies that the point u = ∑i=1^{d+1} t_i x_i is a convex combination of the points in X. Hence, u ∈ conv(X).

Therefore, the ε-ball around v is completely contained in conv(X), which implies that v is an interior point of conv(X). Hence, int(conv(X)) ≠ ∅.

Learn more about independent points here :-

https://brainly.com/question/31987907

#SPJ11

There are 12 points A,B,… in a given plane, no three on the same line. The number of triangles are determined by the points such that contain the point A as a vertex is: (a) 65 (b) 55 (c) 75 (d) 66

Answers

The answer is (c) 75. The number of triangles that can be formed using the points A, B, and C as vertices is 1. We can then choose the remaining vertex from the 9 points that are not A, B, or C. This gives us a total of 9 possible choices for D.

Therefore, the number of triangles that contain A as a vertex is 1 * 9 = 9.

Similarly, we can count the number of triangles that contain B, C, D, E, F, G, H, I, J, K, and L as vertices by considering each point in turn as one of the vertices. For example, to count the number of triangles that contain B as a vertex, we can choose two other points from the 10 remaining points (since we cannot use A or B again), which gives us a total of (10 choose 2) = 45 possible triangles. We can do this for each of the remaining points to get:

Triangles containing A: 9

Triangles containing B: 45

Triangles containing C: 45

Triangles containing D: 36

Triangles containing E: 28

Triangles containing F: 21

Triangles containing G: 15

Triangles containing H: 10

Triangles containing I: 6

Triangles containing J: 3

Triangles containing K: 1

Triangles containing L: 0

The total number of triangles is the sum of these values, which is:

9 + 45 + 45 + 36 + 28 + 21 + 15 + 10 + 6 + 3 + 1 + 0 = 229

However, we have counted each triangle three times (once for each of its vertices). Therefore, the actual number of triangles is 229/3 = 76.33, which is closest to option (c) 75.

Therefore, the answer is (c) 75.

learn more about triangles here

https://brainly.com/question/2773823

#SPJ11

A population of squirrels grows exponentially at a rate of 4.2 percent per year. The population was 8400 in 2002. Step 1 of 3: Find the exponential function that represents the population t years after 2002. Answer Point f(t) =

Answers

Answer:

P(t) = 8,400e^(0.042)t

P(t) = total population t years after 2002

8,400 initial population at 2002

0.042 = rate of growth

t = #years after 2002

Step-by-step explanation:

Using the formula for exponential growth, in this case P(t) = P(subscript 0) e^(kt), k as rate.

P(subscript 0) initial population = 8400

k rate = 4.2% = 0.042

Plug in the numbers as given by the problem.

A fi making toaster ovens finds that the total cost, C(x), of producing x units is given by C(x) = 50x + 310. The revenue, R(x), from selling x units is deteined by the price per unit times the number of units sold, thus R(x) = 60x. Find and interpret (R - C)(64).

Answers

The company makes a profit of $570 by producing and selling 64 units.Given that the cost of producing x units is given by C(x) = 50x + 310 and revenue from selling x units is determined by the price per unit times the number of units sold, thus R(x) = 60x.

To find and interpret (R - C)(64).

Solution:(R - C)(64) = R(64) - C(64)R(x) = 60x, therefore R(64) = 60(64) = $3840.C(x) = 50x + 310, therefore C(64) = 50(64) + 310 = $3270

Hence, (R - C)(64) = R(64) - C(64) = 3840 - 3270 = $570.

Therefore, the company makes a profit of $570 by producing and selling 64 units.

For more question on revenue

https://brainly.com/question/23706629

#SPJ8

Find f′ (2), where f(t)=u(t)⋅v(t),u(2)=⟨2,1,−1⟩,u ′(2)=⟨7,0,6⟩, and v(t)=⟨t,t ^2,t^ 3 ⟩. f ′(2)=

Answers

f'(2) = 56. To find f'(2), we need to use the product rule of differentiation. The product rule states that if we have two functions, u(t) and v(t), then the derivative of their product is given by:

(fg)'(t) = f'(t)g(t) + f(t)g'(t),

where f(t) represents u(t) and g(t) represents v(t).

In this case, we have f(t) = u(t) ⋅ v(t), so we can apply the product rule to find f'(t):

f'(t) = u'(t) ⋅ v(t) + u(t) ⋅ v'(t).

Given:

u(2) = ⟨2, 1, -1⟩,

u'(2) = ⟨7, 0, 6⟩,

v(t) = ⟨t, t^2, t^3⟩.

We can substitute these values into the product rule formula:

f'(t) = u'(t) ⋅ v(t) + u(t) ⋅ v'(t).

f'(2) = u'(2) ⋅ v(2) + u(2) ⋅ v'(2).

Let's calculate each part separately:

u'(2) ⋅ v(2) = ⟨7, 0, 6⟩ ⋅ ⟨2, 4, 8⟩ = 7⋅2 + 0⋅4 + 6⋅8 = 14 + 0 + 48 = 62.

u(2) ⋅ v'(2) = ⟨2, 1, -1⟩ ⋅ ⟨1, 2⋅2, 3⋅2^2⟩ = 2⋅1 + 1⋅4 + (-1)⋅12 = 2 + 4 - 12 = -6.

Finally, we can calculate f'(2) by adding the two results:

f'(2) = u'(2) ⋅ v(2) + u(2) ⋅ v'(2) = 62 + (-6) = 56.

Therefore, f'(2) = 56.

To know more about differentiation visit :

https://brainly.com/question/31539041

#SPJ11

Post Test: Solving Quadratic Equations he tlles to the correct boxes to complete the pairs. Not all tlles will be used. each quadratic equation with its solution set. 2x^(2)-8x+5=0,2x^(2)-10x-3=0,2

Answers

The pairs of quadratic equations with their respective solution sets are:(1) `2x² - 8x + 5 = 0` with solution set `x = {2 ± (sqrt(6))/2}`(2) `2x² - 10x - 3 = 0` with solution set `x = {5 ± sqrt(31)}/2`.

The solution of each quadratic equation with its corresponding equation is given below:Quadratic equation 1: `2x² - 8x + 5 = 0`The quadratic formula for the equation is `x = [-b ± sqrt(b² - 4ac)]/(2a)`Comparing the equation with the standard quadratic form `ax² + bx + c = 0`, we can say that the values of `a`, `b`, and `c` for this equation are `2`, `-8`, and `5`, respectively.Substituting the values in the quadratic formula, we get: `x = [8 ± sqrt((-8)² - 4(2)(5))]/(2*2)`Simplifying the expression, we get: `x = [8 ± sqrt(64 - 40)]/4`So, `x = [8 ± sqrt(24)]/4`Now, simplifying the expression further, we get: `x = [8 ± 2sqrt(6)]/4`Dividing both numerator and denominator by 2, we get: `x = [4 ± sqrt(6)]/2`Simplifying the expression, we get: `x = 2 ± (sqrt(6))/2`Therefore, the solution set for the given quadratic equation is `x = {2 ± (sqrt(6))/2}`Quadratic equation 2: `2x² - 10x - 3 = 0`Comparing the equation with the standard quadratic form `ax² + bx + c = 0`, we can say that the values of `a`, `b`, and `c` for this equation are `2`, `-10`, and `-3`, respectively.We can use either the quadratic formula or factorization method to solve this equation.Using the quadratic formula, we get: `x = [10 ± sqrt((-10)² - 4(2)(-3))]/(2*2)`Simplifying the expression, we get: `x = [10 ± sqrt(124)]/4`Now, simplifying the expression further, we get: `x = [5 ± sqrt(31)]/2`Therefore, the solution set for the given quadratic equation is `x = {5 ± sqrt(31)}/2`Thus, the pairs of quadratic equations with their respective solution sets are:(1) `2x² - 8x + 5 = 0` with solution set `x = {2 ± (sqrt(6))/2}`(2) `2x² - 10x - 3 = 0` with solution set `x = {5 ± sqrt(31)}/2`.

Learn more about equation :

https://brainly.com/question/29657992

#SPJ11

the quotient of 3 and a number m foula r=(d)/(t), where d is the distance in miles, r is the rate, and t is the time in hours, at whic tyou travel to cover 337.5 miles in 4.5 hours? (0pts )55mph (0 pts ) 65mph (1 pt) 75mph X (0 pts ) 85mph

Answers

If the formula r= d/t where d is the distance in miles, r is the rate, and t is the time in hours, you can travel at a rate of 75mph to cover 337.5 miles in 4.5 hours.

To calculate at which rate you travel to cover 337.5 miles in 4.5 hours, follow these steps:

The formula r= d/t, where d is the distance in miles, r is the rate, and t is the time in hours.Substituting the values in the formula, we get r= 337.5/ 4.5= = 75mph.

Therefore, at a rate of 75 miles per hour, you can travel to cover 337.5 miles in 4.5 hours.

Learn more about rate:

brainly.com/question/119866

#SPJ11

∫2+3xdx (Hint: Let U=2+3x And Carefully Handle Absolute Value)

Answers

To evaluate the integral ∫(2+3x)dx, we can use the power rule of integration. However, we need to be careful when handling the absolute value of the expression 2+3x.

Let's first rewrite the expression as U = 2+3x. Now, differentiating both sides with respect to x gives dU = 3dx. Rearranging, we have dx = (1/3)dU.

Substituting these expressions into the original integral, we get ∫(2+3x)dx = ∫U(1/3)dU = (1/3)∫UdU.

Using the power rule of integration, we can integrate U as U^2/2. Thus, the integral becomes (1/3)(U^2/2) + C, where C is the constant of integration.

Finally, substituting back U = 2+3x, we have (1/3)((2+3x)^2/2) + C as the result of the integral.

Learn more about constant of integration here: brainly.com/question/31405248

#SPJ11

There are functions of the form x^{r} that solve the differential equation x²y"-6xy' + 10 y=0
Give the solution to the initial value problem [x²y"-6xy' + 10 y=0 y(1)=0 y'(1)=3]

Answers

The solution in mathematical notation:

y = x² - 1

The differential equation x²y"-6xy' + 10 y=0 is an Euler equation, which means that it can be written in the form αx² y′′ + βxy′ + γ y = 0. The general solution of an Euler equation is of the form y = x^r, where r is a constant to be determined.

In this case, we can write the differential equation as x²(r(r - 1))y + 6xr y + 10y = 0. If we set y = x^r, then this equation becomes x²(r(r - 1) + 6r + 10) = 0. This equation factors as (r + 2)(r - 5) = 0, so the possible values of r are 2 and -5.

The function y = x² satisfies the differential equation, so one solution to the initial value problem is y = x². The other solution is y = x^-5, but this solution is not defined at x = 1. Therefore, the only solution to the initial value problem is y = x².

To find the solution, we can use the initial conditions y(1) = 0 and y'(1) = 3. We have that y(1) = 1² = 1 and y'(1) = 2² = 4. Therefore, the solution to the initial value problem is y = x² - 1.

Here is the solution in mathematical notation:

y = x² - 1

This solution can be verified by substituting it into the differential equation and checking that it satisfies the equation.

Learn more about mathematical notation here:-

https://brainly.com/question/31065745

#SPJ11

Find The General Solution To Y′′+12y′+36y=0.

Answers

Given y′′+12y′+36y=0 We can solve the above second order differential equation by finding the characteristic equation as: r^2 + 12r + 36 = 0

Now, let us find the roots of the above equation: \begin{aligned} r^2 + 6r + 6r + 36 &= 0 \\

\Rightarrow r(r+6) + 6(r+6) &= 0 \\

\Rightarrow (r+6)(r+6) &= 0 \\

\Rightarrow (r+6)^2 &= 0 \end{aligned}

So, we got the repeated roots as r = -6. As the roots are repeated we can write the general solution of the given differential equation as: y(x) = (c_1 + c_2 x) e^{-6x}  

Here c1 and c2 are constants. Hence the general solution of the given second order differential equation is

y(x) = (c1 + c2 x) e^{-6x}.

The given differential equation is y′′+12y′+36y=0.

So, the general solution of the given differential equation is y(x) = (c1 + c2 x) e^{-6x} with c1, c2 being constants.

To know more about equation visit:

https://brainly.com/question/29657983

#SPJ11

a) Let f(x,y) and g(x,y) be Lipschitzian functions. Let h(x,y) be defined by h(x,y)= f(x,y)+g(x,y) and q(x,y) be defined by q(x,y)=αf(x,y), where α is a fixed real number. Prove that h and q are Lipschitzian functions. b) Prove that if f(x,y) and g(x,y) are Lipschitzian functions so is h(x,y) defined by h(x,y)= f(x,g(x,y)).

Answers

h(x, y) is a Lipschitzian function with Lipschitz constant K = K1 * K2.

a) To prove that h(x, y) = f(x, y) + g(x, y) is a Lipschitzian function, we need to show that there exists a constant K such that for any two points (x1, y1) and (x2, y2), the following inequality holds:

| h(x1, y1) - h(x2, y2) | ≤ K * || (x1, y1) - (x2, y2) ||

where || (x1, y1) - (x2, y2) || represents the Euclidean distance between the points (x1, y1) and (x2, y2).

Since f(x, y) and g(x, y) are Lipschitzian functions, we know that there exist constants K1 and K2 such that:

| f(x1, y1) - f(x2, y2) | ≤ K1 * || (x1, y1) - (x2, y2) ||  ... (1)

| g(x1, y1) - g(x2, y2) | ≤ K2 * || (x1, y1) - (x2, y2) ||  ... (2)

Now, let's consider the difference h(x1, y1) - h(x2, y2):

h(x1, y1) - h(x2, y2) = [f(x1, y1) + g(x1, y1)] - [f(x2, y2) + g(x2, y2)]

                     = [f(x1, y1) - f(x2, y2)] + [g(x1, y1) - g(x2, y2)]

Using the triangle inequality, we have:

| h(x1, y1) - h(x2, y2) | ≤ | f(x1, y1) - f(x2, y2) | + | g(x1, y1) - g(x2, y2) |

Applying inequalities (1) and (2), we get:

| h(x1, y1) - h(x2, y2) | ≤ K1 * || (x1, y1) - (x2, y2) || + K2 * || (x1, y1) - (x2, y2) ||

Since K = K1 + K2, we can rewrite the above inequality as:

| h(x1, y1) - h(x2, y2) | ≤ K * || (x1, y1) - (x2, y2) ||

Therefore, h(x, y) is a Lipschitzian function with Lipschitz constant K.

b) To prove that h(x, y) = f(x, g(x, y)) is a Lipschitzian function, we need to show that there exists a constant K such that for any two points (x1, y1) and (x2, y2), the following inequality holds:

| h(x1, y1) - h(x2, y2) | ≤ K * || (x1, y1) - (x2, y2) ||

Let's consider the difference h(x1, y1) - h(x2, y2):

h(x1, y1) - h(x2, y2) = f(x1, g(x1, y1)) - f(x2, g(x2, y2))

Since f(x, y) is a Lipschitzian function, we know that there exists a constant K1 such that:

|

f(x1, g(x1, y1)) - f(x2, g(x2, y2)) | ≤ K1 * || (x1, g(x1, y1)) - (x2, g(x2, y2)) ||

Now, let's consider the distance || (x1, y1) - (x2, y2) ||:

|| (x1, y1) - (x2, y2) || = || (x1, g(x1, y1)) - (x2, g(x2, y2)) ||

Since g(x, y) is a Lipschitzian function, we know that there exists a constant K2 such that:

|| (x1, g(x1, y1)) - (x2, g(x2, y2)) || ≤ K2 * || (x1, y1) - (x2, y2) ||

Combining these inequalities, we have:

| h(x1, y1) - h(x2, y2) | ≤ K1 * || (x1, g(x1, y1)) - (x2, g(x2, y2)) || ≤ K1 * K2 * || (x1, y1) - (x2, y2) ||

Learn more about Lipschitzian function here :-

https://brainly.com/question/29249394

#SPJ11

What is the missing reason in the proof?
Given: AABE ACDE
Prove: ABCD is a parallelogram.
Statement
1. AABE is congruent to ACDE.
2. BE is congruent to DE
AE is congruent to CE.
3. AC and BD bisect each other.
4. ABCD is a parallelogram.
A. Opposite sides property
B. CPCTC
Reason
Given
CPCTC
?
Converse of diagonals theorem

Answers

The missing reason in the proof is determined as the:

converse of the diagonals theorem

What is the Converse of diagonals theorem?

The converse of the diagonals theorem states that if the diagonals of a quadrilateral bisect each other, then the quadrilateral is a parallelogram.

The proof is shown in the image attached below, which shows that in step 3, the diagonals of the quadrilateral bisect each other. Therefore, based on the converse of the diagonals theorem, we can conclude that the quadrilateral is a parallelogram.

Learn more about  Converse of diagonals theorem on:

https://brainly.com/question/17142779

#SPJ1

Find The Area Bounded By The First Quadrant Loop Of The Curve X^5+Y^5=3xy

Answers

The area bounded by the first quadrant loop of the curve x^5 + y^5 = 3xy is approximately 0.536 square units.

To find the area bounded by the curve x^5 + y^5 = 3xy in the first quadrant, we can use the double integral. However, this particular curve is quite complicated to work with directly. Instead, we can use a change of variables to simplify the equation.

Let's make the substitution u = x^5 and v = y^5. Then, we can express the curve equation in terms of u and v:

u + v = 3uv

This is a much simpler equation to work with. Now, let's find the limits of integration for u and v. Since we are considering the first quadrant, both u and v must be positive. From the original equation, we can see that when x = 0, y = 0, and when y = 0, x = 0. Therefore, the limits of integration for u and v are both from 0 to 1.

Now, we can calculate the area using the double integral:

A = ∬R dA

A = ∫∫R du dv

A = ∫[0,1] ∫[0,1] du dv

A = ∫[0,1] u=0 to 1 v=0 to 1 du dv

A = ∫[0,1] (v/2 + v^2/3) u=0 to 1 dv

A = ∫[0,1] (1/2 + v/3) dv

A = (1/2)v + (1/6)v^2 from 0 to 1

A = (1/2)(1) + (1/6)(1^2) - (1/2)(0) - (1/6)(0^2)

A = 1/2 + 1/6

A = 3/6 + 1/6

A = 4/6

A ≈ 0.667 square units

Therefore, the area bounded by the first quadrant loop of the curve x^5 + y^5 = 3xy is approximately 0.667 square units.

Learn more about curve equation here:

brainly.com/question/31467851

#SPJ11

"
Gym A charges $18 per month plus a $25 fee. Gym B charges $6 per month plus a $97 fee. a. Gym A and B will cost the same at _________________________ months. b. How much will it cost at that time?
"

Answers

a. Gym A and B will cost the same at 11 months.

b. It will cost $223.00 at that time.

Let's calculate the cost of each gym and find out the time at which both gyms will cost the same.

Gym A cost = $18 per month + $25 fee

Gym B cost = $6 per month + $97 fee

Let's find out when the costs of Gym A and Gym B will be the same.18x + 25 = 6x + 97   (where x represents the number of months)18x - 6x = 97 - 2512x = 72x = 6Therefore, Gym A and Gym B will cost the same after 6 months.

Let's put x = 11 months to calculate the cost of both gyms at that time.

Cost of Gym A = 18(11) + 25 = $223.00Cost of Gym B = 6(11) + 97 = $223.00

Therefore, it will cost $223.00 for both gyms at 11 months.

Learn more about cost:https://brainly.com/question/28147009

#SPJ11

Calculate the margin of error and construct a confidence interval for the population proportion using the normal approximation to the p^ -distribution (if it is appropriate to do so). a. p^=0.8,n=120,α=0.02 E= Round to four decimal places if necessary Enter o if normal approximation cannot be used

Answers

The margin of error (E) can be calculated using the formula [tex]E = z_{\frac{\alpha}{2}}\sqrt{\frac{p^*(1-p^*)}{n}}$[/tex], where [tex]z_{\frac{\alpha}{2}}$[/tex] is the z-value with a cumulative probability of -2.33. Using the standard normal distribution table, the z-value corresponding to 0.01 is -2.33. The margin of error (E) is 0.0736, allowing for a 95% confidence interval for the true population proportion (p) using the normal approximation to the binomial distribution.

The formula to calculate the margin of error in this case is given by the formula below: [tex]$E = z_{\frac{\alpha}{2}}\sqrt{\frac{p^*(1-p^*)}{n}}$[/tex],

where [tex]$z_{\frac{\alpha}{2}}$[/tex] is the z-value with a cumulative probability of [tex]$\frac{\alpha}{2}$, $p^*$[/tex]

is the sample proportion, and n is the sample size. Now, given that p^ = 0.8, n = 120 and α = 0.02, we can calculate the margin of error (E) as follows:

Firstly, we need to find the z-value with a cumulative probability of

[tex]$\frac{\alpha}{2}$ or $\frac{0.02}{2}[/tex] = 0.01

in the standard normal distribution table. The z-value corresponding to 0.01 is -2.33. Then, substituting these values into the formula above we get:

[tex]$$E = z_{\frac{\alpha}{2}}\sqrt{\frac{p^*(1-p^*)}{n}} = -2.33\sqrt{\frac{0.8(1-0.8)}{120}}$$ $$E = 0.0736$$[/tex]

Therefore, the margin of error (E) is 0.0736. This means that we can construct a confidence interval for the true population proportion (p) with 95% confidence using the formula below[tex]:$$CI = \left(p^ - E, p^ + E \right)$$[/tex] Where p^ is the sample proportion. Now substituting the values given above we get:[tex]$$CI = \left(0.8 - 0.0736, 0.8 + 0.0736 \right)$$ $$CI = (0.7264, 0.8736)$$[/tex]

Hence, the 95% confidence interval for the true population proportion (p) is (0.7264, 0.8736). We used the normal approximation to the binomial distribution since the sample size is large enough.

To know more about margin of error Visit:

https://brainly.com/question/29419047

#SPJ11

Cos(x), where x is in radians, can be defined by the following infinite series: cos(x)=∑ n=0
[infinity]

(2n)!
(−1) n
x 2n

=1− 2!
x 2

+ 4!
x 4

− 6!
x 6

+ 8!
x 8

+⋯ Carry your answers for parts a,b, and c below to six decimal places. x= 4
π

a) What is the value of cos(π/4) if the series is carried to three terms? b) What is the value of cos(π/4) if the series is carried to four terms? c) What is the approximate absolute error, E A

, for your estimation of cos(π/4) ? d) What is the approximate relative error, ε A

, for your estimation, as a percentage? Carry this answer to 3 significant figures. 3.14 The velocity of a flow may be measured using a manometer, a pitot-static tube, and the following formula: V= rho
2∗γ∗h


where γ is the specific weight of the manometer fluid, h is the differential height in the manometer legs, and rho is the density of the flowing fluid. Given γ=57.0±0.15lb/ft 3
,h=0.15±0.01ft, and rho=0.00238 ±0.0001slug/ft 3
, determine the speed of the flow and its uncertainty. Perform both exact and approximate analyses and present your answers in absolute and relative form.

Answers

The value of cos(π/4) when the series is carried to three terms is 0.707107, the value of cos(π/4) when the series is carried to four terms is 0.707103 and the approximate relative error for the estimation of cos(π/4) is 0.000565%.

a) To find the value of cos(π/4) using the series expansion, we can substitute x = π/4 into the series and evaluate it to three terms:

cos(π/4) = 1 - (2!/(π/4)^2) + (4!/(π/4)^4)

Calculating each term:

2! = 2

(π/4)^2 = (3.14159/4)^2 = 0.61685

4! = 24

(π/4)^4 = (3.14159/4)^4 = 0.09663

Now, plugging the values into the series:

cos(π/4) ≈ 1 - 2(0.61685) + 24(0.09663) = 0.707107

Therefore, the value of cos(π/4) when the series is carried to three terms is approximately 0.707107.

b) To find the value of cos(π/4) using the series expansion carried to four terms, we include one more term in the calculation:

cos(π/4) ≈ 1 - 2(0.61685) + 24(0.09663) - ...

Calculating the next term:

6! = 720

(π/4)^6 = (3.14159/4)^6 = 0.01519

Now, plugging the values into the series:

cos(π/4) ≈ 1 - 2(0.61685) + 24(0.09663) - 720(0.01519) = 0.707103

Therefore, the value of cos(π/4) when the series is carried to four terms is approximately 0.707103.

c) The approximate absolute error, EA, for the estimation of cos(π/4) can be calculated by comparing the result obtained in part b with the actual value of cos(π/4), which is √2/2 ≈ 0.707107.

EA = |0.707107 - 0.707103| ≈ 0.000004

Therefore, the approximate absolute error for the estimation of cos(π/4) is approximately 0.000004.

d) The approximate relative error, εA, for the estimation can be calculated by dividing the absolute error (EA) by the actual value of cos(π/4) and multiplying by 100 to express it as a percentage.

εA = (EA / 0.707107) * 100 ≈ (0.000004 / 0.707107) * 100 ≈ 0.000565%

Therefore, the approximate relative error for the estimation of cos(π/4) is approximately 0.000565%.

To know more about relative error, visit:

https://brainly.com/question/30403282#

#SPJ11

Other Questions
Select ALL that apply. Which of the following would be helpful in reducing greenhouse gas emissions?Building more efficient internal combustion vehicles, but using them more.Making energy from clean sources affordable and cheaper than subsidized fossil fuels.Increasing consumption of alternative meat proteins such as insects.Decreasing the connectivity within our cities and increasing urban sprawl.Making efforts to restore natural ecosystems and improving soil fertility.Incorporating more telecommunication, tele-education and virtual entertainment in our lives.Diverting finances from fossil fuel subsidies to support public expenditures used to expand social safety nets. an accrual method corporation, produced a rock concert on december 28, 2022. gross reccipts were $400,000 and expenses were as follows: (i) cost of good sold $90,000, (it) cost of performers $100,000 and (ili) $75,000 cost of cleaning up the venuc which occurred on january 20, 2023. prince co. purchased the goods sold prior to december 28, paid the performers on the night of december 28 and paid the maintenance/clean up crew on january 20,2023. what is prince co. net income for tax purposes (from the concert) reported on its 2022 income tax return? how did south africa achieve majority rule? south africa achieved majority rule through a process of mediation led by nongovernmental organizations (ngos), followed by free elections. Entrepreneurial Trends in 2022/2023 this discussion bosard. 6 replies, 6 unread [ Discussion: 16 Entrepreneurial Trends in 2022/2023 Post a meaninglul, multrrentence (200 word minimumh, college level comment to "16 Entrepreneurial Trends for 2022/2023" discussion board wagetsting at least one trend that innt on the list that you feel SHOULD be and why? Afe there trends happening on the college campus? Within your work environment' On the home front? (Be specifc). In addition, post a meaningful, multi-sentence response to one of your peers' posts in this discussion board. Tdit View inen format Tools Table Suppose the scores, X, on a college entrance examination are normally distributed with a mean of 1000 and a standard deviation of 100 . If you pick 4 test scores at random, what is the probability that at least one of the test score is more than 1070 ? The total cost (in dollars) of producing x coffee machines is C(x)=2500+70x0.3x2 (A) Find the exact cost of producing the 21st machine. Exact cost of 21 st machine = (B) Use marginal cost to approximate the cost of producing the 21 st machine. Approx. cost of 21 st machine = Required information [The following information applies to the questions displayed below.] Del Gato Clinic's cash account shows a $11,557 debit balance and its bank statement shows $10,864 on deposit at the close of business on June 30 . a. Outstanding checks as of June 30 total $1,522. b. The June 30 bank statement lists a $20 bank service charge. c. Check No. 919, listed with the canceled checks, was correctly drawn for $389 in payment of a utility bill on June 15. Del Gato Clinic mistakenly recorded it with a debit to Utilities Expense and a credit to Cash in the amount of $398. d. The June 30 cash receipts of $2,204 were placed in the bank's night depository after banking hours and were not recorded on the June 30 bank statement. Prepare its bank reconciliation using the above information. Solve each of the following initial value problems and plot the solutions for several values of yo. Then describe in a few words how the solutions resemble, and differ from, each other. a. dy/dt=-y+5, y(0) = 30 b. dy/dt=-2y+5, y(0) = yo c. dy/dt=-2y+10, y(0) = yo Which of the following are advantages of English as a global lingua franca? (Be sure to click all that apply. There are multiple answers to this question.)- Everyone will get along better because they speak the same language.- Communications in international organizations like the Association of South-East Asian Nations is easier.- International travel is more accessible.- People for whom English is an L1 (ex: people in Great Britain and the US) don't have to learn more than one language.- English is sophisticated and easy to learn compared to other potential lingua francas, for instance Chinese. step by step please: calculate the differential equation a. dx/dt+7x = 5cos2t using first-order differential equation Graph the system of equations on graph paper to answer the question. {y=13x2y=3x12 What is the solution for this system of equations? Enter your answer in the boxes.( , ) Which is larger, 0. 45 mol of a material or 2. 75 x 10% of the same material? Justify your answer Which of the following is an expected finding in patients who have a left ventricular assist device (LVAD) with a continuous flow pump?A) cyanotic skin.B) hypertension.C) peripheral edema.D) absence of pulses. On January 1, a fund is worth 100, 000. On June 1, the value has increasedto 120, 000 and then 30, 000 of new principal is deposited. On October 1, thevalue has declined to 130, 000 and then 50, 000 is withdrawn. On January 1of the following year, the fund is again worth 100, 000. Calculate the dollar-weighted rate of return using the simple interest approximation. as a part of her studies, jolyn gathered data on the length of time between dentist visits for a sample of 23 individuals. she works through the testing procedure: Is an unreliable narrator biased? Bondseller Inc. has a December 31 fiscal year end. On January 1,2021 , Bondseller Inc, issued bonds with a face value of $20,000,000. The bonds have a coupon rate of 8% and mature on December 31,2025 . The bonds pay interest semiannually on June 30 and December 31 each year. At the time the bonds were issued, the market rate of interest for similar bonds was 6\%. Given that 011001101458 100010001514write 458as a linear combination of the vectors 011, 001, 101458= 011+ 001+ 101Problem 9. Describe the set of all matrices that are row equivalent to [ 100000] the type of state securities registration for an initial public offering that relies on the prospectus info filed with the SEC under federal law is registration by because he seems to be easily persuaded by his friends to do things out of concern to fit in with them. this persuasion is known as